Private School - Elementary vs Middle/High (bang for the buck)

Questions on how we spend our money and our time - consumer goods and services, home and vehicle, leisure and recreational activities
Topic Author
sdsu04
Posts: 83
Joined: Sun Nov 19, 2017 10:53 pm

Private School - Elementary vs Middle/High (bang for the buck)

Post by sdsu04 »

Hello!

Hoping to get some feedback on the value for private school education for elementary school children vs spending on private school during later academic years (Middle/high school).

While we (my SO and I) evaluate possible new opportunities that require us to move to a new city, we may be faced with the option of spending on a pricier home (Bay area / Pacific Northwest standards) with a good school district vs a comparatively lower priced home where the public schools are poor such that the kids would need to go to a private school. Understanding the below helps us better plan this out financially (knowing how much more we would spend for education). Currently we have 2 kids and one is in elementary school and the other is still in day care.

Question:

1. Is it better to invest In a Private school education during elementary school years - which are formative years for the child?

2. Or is there minimal difference (in value) between private and public school when the child is still in elementary. In other words, private schools are a better investment later in life?

3. If a child does not enter private school while in elementary, would it be harder to find a spot during later years (this was the case when I went to school outside the US).

4. Do private schools prepare the child and provide a better opportunity to a higher ranked college (or a college of their choice)? Understand this is not a yes/no answer but your experience would be helpful to learn about and help us better understand their value.

5. Finally, for those who opted to send their child to a private school, while still having access to one of the best public schools - what was the reason?

Thanks for your inputs!
User avatar
lthenderson
Posts: 8528
Joined: Tue Feb 21, 2012 11:43 am
Location: Iowa

Re: Private School - Elementary vs Middle/High (bang for the buck)

Post by lthenderson »

Not sure I can answer your questions but we chose to send both our kids through elementary years at a private school. When the oldest graduated from middle school she was very advanced to her peers and graduated in the top 5%. In high school she is still advanced and taking all AP classes. So I feel at least that the private elementary school was able to give her a head start that is still helping her stay ahead. The youngest is half way through private elementary school and according to standardized testing is also ahead of her peers nationally so I'm hoping the same with her.

One drawback was that when she entered the public school stream, she was a bit socially stunted as most of her new classmates had been together in the public school system for six years at that point.

In our area, there is no option for private middle or high school so we didn't have to decide for later years.
User avatar
leeks
Posts: 1466
Joined: Thu Apr 07, 2011 4:33 pm
Location: virginia

Re: Private School - Elementary vs Middle/High (bang for the buck)

Post by leeks »

It depends on the child, depends on the public school, depends on the private school. Different cities and neighborhoods have very different school dynamics.

There is really no way to get a general answer to this.

An anecdote only - we are trying a private Montessori elementary next year for our children entering K and 2nd grades. It is not a status symbol to us - in fact we are somewhat embarrassed/ashamed to admit to friends and family that our kids will attend a private. It violates what we thought were our financial and social values. We are the products of public schools ourselves (spoiler alert: today's version is not the same) and have a number of public school teachers in our families and social networks. Actually some of those public school teachers also send at least one of their kids to private school.

We gave public school a try (in a different city than where we are relocating). A desirable gifted and talented program at an 8/10 (in those silly online ratings that aren't that helpful) public elementary was far too easy and boring for our child and even the pre-pandemic environment was stressful to him (mostly sit down all day, almost no play, little outdoor time, little movement, not enough time to eat lunch, etc).

A child who is advanced and/or well prepared at home - who can read fluently and do basic addition/subtraction when entering K - will be bored for *years* in the public curriculum we tried and others we investigated. For some children (especially my highly sensitive one), this can lead to misbehavior or learning to hate school. The curriculum lacked both depth and breadth - even before the pandemic version made it truly intolerable (we unenrolled him last fall to homeschool instead). Further lowering my opinion of public school, I found the online zoom+youtube and then "hybrid" school to be unforgivably inappropriate for young children (while most privates were fully in-person!).

There is a push for public schools to meet minimum test scores for reading/math in 3rd grade and so prior to that there is little or no science, history, literature, civics, geography, foreign language, etc. They just weren't teaching any particular facts - all the instructional time was focused on basic reading and math skills my child already had. They used the Teacher's College reading and writing workshop stuff for hours each day and it seemed like useless mumbo jumbo waste-of-time to me (but a lot of other people think it is high quality). Also they are opposed to directly teaching any grammar or spelling.

We have investigated public school curricula in other east coast cities all in walkable urban/inner-suburb neighborhoods (my husband would never move to car-dependent suburbs) and found nothing tolerable in the early elementary years, except maybe some lottery-based magnet programs where buying a house does not guarantee access.

We are trying private to get much smaller student:teacher ratio (my older kid loves talking to a grown up), mixed-age classrooms (great for children whose development is asynchronous), beautiful surroundings and hands-on materials and campus with gardens (good for sensory stuff and reducing stress), and a curriculum that covers a good range of subjects and is individualized so children work at their own pace and can be adequately challenged.

The switch to private elementary is for our children's emotional health more than anything else - we want them to be happy, not stressed, and to continue to have a love of learning. And we do not want to homeschool long term.

We hope by middle and high school there will be adequate public options once there is more differentiation (ie programs for advanced kids).

But, again, the areas we considered are not reflective of the country as a whole. And our children are not "normal" if there is such a thing. Apparently, neither are my own education standards.
Last edited by leeks on Wed Jun 16, 2021 12:35 am, edited 3 times in total.
runner3081
Posts: 5994
Joined: Mon Aug 22, 2016 3:22 pm

Re: Private School - Elementary vs Middle/High (bang for the buck)

Post by runner3081 »

So much of this depends on your kid, types of schools around, other kids in the class, etc.

For us, we didn't consider private schools for a moment. Plenty of people around us put their kids in private schools. Doesn't seem to end up being worth it to them, outside of saying, "our kid goes to (insert prestigious private school name) here.
cbs2002
Posts: 698
Joined: Thu Feb 27, 2020 1:10 pm

Re: Private School - Elementary vs Middle/High (bang for the buck)

Post by cbs2002 »

private elementary school parent here. A quick search tells me that 90% of U.S. children attend public schools. Many many of them become highly competent and successful members of society.

It's impossible to generalize about "bang for the buck". You could evaluate it based on these criteria, maybe, which are the reasons I observe people sending kids to private school:

1) access to elite networks in an attempt to maintain or create access to future professional and social options
2) the parents want to live in a place that has subpar public schools
3) the child has a genuine need for some service or opportunity that is impossible to get in a public school setting
4) the parents want some control over the child's experience that they cannot get from public schools (e.g. religion, culture, access to staff, class size, teacher qualification, etc)

My advice would be to know what reason(s) you are choosing private over public, and be honest with yourself about it, rather than following a vague feeling that somehow paying for education (outside of the property taxes you will already pay, I mean) will provide a better life for your child. Cause that is not the case.

Our experience has been both positive and negative. The positives were in areas that we knew were specific needs for one of our children, and the school we chose addressed those needs. I hope to go public in the future.
Simple Simon
Posts: 166
Joined: Sat Apr 26, 2014 4:37 am

Re: Private School - Elementary vs Middle/High (bang for the buck)

Post by Simple Simon »

We were in your situation.

We opted for private school from the earliest age. We were not sure if we would be able to afford it later.

But things have a way of working out when you need them to. We made sure we earned enough to see it through.

It was a good choice for us, government schools would not have worked so well for our family.

No regrets here.

On the other hand, we are still working. Could be retired if we had not had this expense.
25% stock, 25% bonds, 25% cash, 25% stuff
rascott
Posts: 2957
Joined: Wed Apr 15, 2015 10:53 am

Re: Private School - Elementary vs Middle/High (bang for the buck)

Post by rascott »

This is an impossible question to answer.

Nobody here knows your specific area and what makes sense for your family.

Find a school that a house and school that fits in your budget. Kids will do fine in most places. They are very adaptable. Adults are usually the problem.
DonFifer
Posts: 85
Joined: Wed May 05, 2021 1:59 pm

Re: Private School - Elementary vs Middle/High (bang for the buck)

Post by DonFifer »

I can only relate a personal experience with Parochial Schooling. My two sisters, brother and I were all brought up through parochial schools. In my brother's junior year in high school, they wanted to hold him back a year. He pleaded with my parents to let him attend public school. The public school did not hold him back a year, in fact, he only had to take two shop classes to graduate. For him, his senior was one giant party.
User avatar
winterfan
Posts: 826
Joined: Mon Jan 05, 2015 10:06 am

Re: Private School - Elementary vs Middle/High (bang for the buck)

Post by winterfan »

In some cities where private school admission is competitive, then yes, it's usually better if they are in an elementary/MS feeder school. As far as bang for the buck, I'm not sure. It depends on too many factors (your area, your child's needs, class size, academic rigor, etc.).
MMiroir
Posts: 852
Joined: Sun Mar 29, 2020 12:14 pm

Re: Private School - Elementary vs Middle/High (bang for the buck)

Post by MMiroir »

leeks wrote: Tue Jun 15, 2021 5:57 pm It depends on the child, depends on the public school, depends on the private school. Different cities and neighborhoods have very different school dynamics.

There is really no way to get a general answer to this.
To expand on that, it depends in how many kids one has, family income levels, how gifted the kids are and what the parents expectations are. Our experience when our kids were younger was that public schools in the city we lived in were terrible, and the privates were very expensive and very competitive to get into. We decided to move out to the suburbs, and pay more in home price and real estate tax to live in a community with excellent public schools. It paid off very well now that our kids are off to college, but had we lived in a different geographic location, we might have made different choices as excellent quality public schools are not available in many areas.
Frank the Tank
Posts: 71
Joined: Fri Oct 16, 2020 9:24 am

Re: Private School - Elementary vs Middle/High (bang for the buck)

Post by Frank the Tank »

There's no one size fits all answer because there are too many variables and it's way too broad to make a "public vs. private" comparison.

Some private schools are highly competitive while others are essentially open admissions as long as you can write a check.

Some public schools are going to be as high quality as any private school while others are going to present issues.

Where we live, the public school districts are very highly rated, so I personally would find it to be a horrible "bang for your buck" to pay for private schools on top of our property taxes (and I outright believe that our children are receiving a better education in our public schools regardless of cost).

If we lived 5 miles away in a different public school district that's much lower rated, the calculus for us would probably be totally different.

With the way college admissions work these days, I personally wouldn't be basing any public/private school decision on any perceived advantage to get into higher-rated colleges. There might be a very small handful of super elite private schools that provide some material advantages (e.g. Philips Exeter, Harvard-Westlake), but a typical upper middle class private high school generally isn't going to have any discernible college admissions advantage over a typical upper middle class public high school. As long as a local public high school provides a full slate of rigorous classes for high performers (e.g. AP classes), it's all about what your kid is able to achieve as opposed to the school itself.

You mentioned that you went to school outside of the US, so please note that elite colleges such as the Ivy League universities focus much more on individual "hooks" since they receive way more applicants with perfect test score/perfect grades than they have spaces for in their classes. Getting into an Ivy League school (or similarly situated ones like Stanford, MIT, etc.) is NOT an objective measure of academic achievement like it is in many (most?) other countries. For instance, 20-25% of each Ivy League class is dedicated to recruited athletes. Frankly, at a typical suburban high school these days (public or private), you're more likely to find a very smart kid that's a superstar at baseball or another sport (e.g. good enough that they'd be getting a Division I athletic scholarship at another university) get into the Ivy League than just a "normal" valedictorian with a perfect academic record.

Another significant percentage is allocated to legacy admissions (children of alumni) - those kids usually have the grades/test scores to get admitted, so they're getting a preference over non-legacy applicants with similar stats. There are also development cases (children of super-high donors). A lot of wealthy high schools may have a larger number of legacy and/or development applicants, so that can skew the numbers (where an applicant really got in because of legacy/development status as opposed to the high school itself). The number of "unhooked" applicants at elite universities is much lower than people think. To be sure, it doesn't mean that it's impossible for a "normal" valedictorian to get into those colleges, but rather it's not a transparent or completely objective process (which is difficult for people that come from countries to understand where college admissions might be based solely on grades and/or test scores) and they're competing for a smaller number of slots compared to recruited athletes (who have virtually guaranteed admissions).

This is a long-winded way of saying that you should find the best educational situation for your children now and don't fool yourself into thinking that one particular school is going to provide a material advantage for getting into colleges compared to another one (assuming that a school isn't clearly deficient, e.g. low-performing, doesn't offer AP classes, etc.).
MMiroir
Posts: 852
Joined: Sun Mar 29, 2020 12:14 pm

Re: Private School - Elementary vs Middle/High (bang for the buck)

Post by MMiroir »

To answer some of these questions more specifically:
sdsu04 wrote: Tue Jun 15, 2021 5:20 pm1. Is it better to invest In a Private school education during elementary school years - which are formative years for the child?
It depends on the school, but I would try to find an elementary school that had a gifted/accelerated program. I will say that our highly rated elementary school had a gifted/accelerated program, but that kids that were enrolled in it were not necessarily the most successful academically in high school.
sdsu04 wrote: Tue Jun 15, 2021 5:20 pm4. Do private schools prepare the child and provide a better opportunity to a higher ranked college (or a college of their choice)? Understand this is not a yes/no answer but your experience would be helpful to learn about and help us better understand their value.
It depends. If you are talking about going to a regional public university vs the state flagship, it probably does not make a difference. If you are talking about good private university or top public outside the top 10 or 20, the private might offer better placement as they generally have better/more numerous counselors, and the student body may have more expectation of attending a higher ranked college and the willingness to pay for it. However, that will vary from private to private.

For the top 20 schools, and particularly the Ivies, they may be no benefit, and attending a top private may be actually be a detriment to admission to one of these types of schools.
sdsu04 wrote: Tue Jun 15, 2021 5:20 pm5. Finally, for those who opted to send their child to a private school, while still having access to one of the best public schools - what was the reason?
We live in a top public school district, and know a few parents that sent their kids to private school. The reasons were generally not related to college placement, but were as follows:

1 – Religious – This should be self-explanatory.

2 – Social – Their kids had a bad experience in the public-school social scene, and they wanted to separate their kids from an existing acquaintance group due to drugs, bullying etc. Sending them to a local private cut their kids off from people who were not good influences.

3 – Sports – The kids wanted to play a varsity sport, and it is much less competitive to play varsity in a smaller school.

4 – Security – Family owns or manages a major employer, and they don’t want their kids to attend the same school as those of their employees. In case of layoffs, firings etc., they don’t their kids to be at risk of retribution from angry classmates.

5 – Special Needs – Some privates are better suited to dealing with children with emotional or development needs. One friend of ours had a daughter with some severe emotional issues in elementary school, and the parents decided to send her to a private out-of-state boarding military academy. The structure worked wonders for her as she ended up at West Point for college and is now a commissioned officer.
Isabelle77
Posts: 958
Joined: Thu Dec 17, 2015 12:43 pm

Re: Private School - Elementary vs Middle/High (bang for the buck)

Post by Isabelle77 »

In addition to the lists above in my state is a general anger at the way the last year was handled. We didn’t even consider the public magnet program we had been considering for our son. The private schools are all full where we live in Washington, not a usual thing.
Hoosier CPA
Posts: 512
Joined: Tue Nov 22, 2016 8:43 pm

Re: Private School - Elementary vs Middle/High (bang for the buck)

Post by Hoosier CPA »

Our kids just finished 3rd, 3rd and 6th grade. For us, it's about developing a worldview from a religious perspective. We're not sure how long we will be able to afford it, but we decided we would prefer to start early as the early years are the foundation. From that perspective we thought the earlier years were more important for us. This isn't an ROI decision for us.

The school our children attend is challenging academically, but that's not the key factor for us, as our local public schools are highly regarded.
HereToLearn
Posts: 1537
Joined: Sat Mar 17, 2018 5:53 pm

Re: Private School - Elementary vs Middle/High (bang for the buck)

Post by HereToLearn »

Frank the Tank wrote: Wed Jun 16, 2021 11:03 am There's no one size fits all answer because there are too many variables and it's way too broad to make a "public vs. private" comparison.

Some private schools are highly competitive while others are essentially open admissions as long as you can write a check.

Some public schools are going to be as high quality as any private school while others are going to present issues.

Where we live, the public school districts are very highly rated, so I personally would find it to be a horrible "bang for your buck" to pay for private schools on top of our property taxes (and I outright believe that our children are receiving a better education in our public schools regardless of cost).

If we lived 5 miles away in a different public school district that's much lower rated, the calculus for us would probably be totally different.

With the way college admissions work these days, I personally wouldn't be basing any public/private school decision on any perceived advantage to get into higher-rated colleges. There might be a very small handful of super elite private schools that provide some material advantages (e.g. Philips Exeter, Harvard-Westlake), but a typical upper middle class private high school generally isn't going to have any discernible college admissions advantage over a typical upper middle class public high school. As long as a local public high school provides a full slate of rigorous classes for high performers (e.g. AP classes), it's all about what your kid is able to achieve as opposed to the school itself.

You mentioned that you went to school outside of the US, so please note that elite colleges such as the Ivy League universities focus much more on individual "hooks" since they receive way more applicants with perfect test score/perfect grades than they have spaces for in their classes. Getting into an Ivy League school (or similarly situated ones like Stanford, MIT, etc.) is NOT an objective measure of academic achievement like it is in many (most?) other countries. For instance, 20-25% of each Ivy League class is dedicated to recruited athletes. Frankly, at a typical suburban high school these days (public or private), you're more likely to find a very smart kid that's a superstar at baseball or another sport (e.g. good enough that they'd be getting a Division I athletic scholarship at another university) get into the Ivy League than just a "normal" valedictorian with a perfect academic record.

Another significant percentage is allocated to legacy admissions (children of alumni) - those kids usually have the grades/test scores to get admitted, so they're getting a preference over non-legacy applicants with similar stats. There are also development cases (children of super-high donors). A lot of wealthy high schools may have a larger number of legacy and/or development applicants, so that can skew the numbers (where an applicant really got in because of legacy/development status as opposed to the high school itself). The number of "unhooked" applicants at elite universities is much lower than people think. To be sure, it doesn't mean that it's impossible for a "normal" valedictorian to get into those colleges, but rather it's not a transparent or completely objective process (which is difficult for people that come from countries to understand where college admissions might be based solely on grades and/or test scores) and they're competing for a smaller number of slots compared to recruited athletes (who have virtually guaranteed admissions).

This is a long-winded way of saying that you should find the best educational situation for your children now and don't fool yourself into thinking that one particular school is going to provide a material advantage for getting into colleges compared to another one (assuming that a school isn't clearly deficient, e.g. low-performing, doesn't offer AP classes, etc.).
I agree with all of the above, other than one slight point of clarification. Recruited athletes represent more of the first year class at say Dartmouth than at Penn or Cornell. The Ivies are all subject to the same recruiting agreements--I think it is still called Academic Index for Athletic Recruiting--but my info may be out of date. If I recall correctly, each school recruits 200-250 athletes each year, so a higher percentage at small Dartmouth vs larger Cornell.

Some of the athletic admits are also legacy admits. Fewer development admits than we all think.

Agree with your comment 100%, and will add as long as there is a like-minded peer group: "As long as a local public high school provides a full slate of rigorous classes for high performers (e.g. AP classes), it's all about what your kid is able to achieve as opposed to the school itself."

My children attended public HS but we live in an area with a number of private schools, and a decent # of students who go away to boarding school. Public school grads' outcomes were absolutely comparable.
fwellimort
Posts: 890
Joined: Tue Feb 12, 2019 8:41 am

Re: Private School - Elementary vs Middle/High (bang for the buck)

Post by fwellimort »

I attended both public and private during my pre-college academic years.
I think I ended up pretty well academically considering I ended up graduating from Columbia Univ in City of NY.

From my experience, there's not much of a difference between a decent public and a good private school.
Sure the good private school has better academics and all but I felt the differences were pretty minute and not worth the additional costs.

What I felt were of the greatest help academically were:
* Having my parents expose me to lego robotics (exposes to basic engineering and programming) through FIRST Robotics and all
* Building habit of reading books regularly (Harry Potter, Twilight, Eragon, Artemis Fowl, Twilight, Percy Jackson, etc.)
* Building strong foundation of mathematics through doing lots of arithmetic early on and then doing the Art of Problem Solving math textbooks (these are VERY difficult and I wouldn't expect even college graduates to do all the questions from even Pre-Algebra have they never been exposed to competition math).
AoPS books are THE math books to do if you want a really solid foundation in math. In my mind, the best math books. If you can do all the AoPS books, then I expect you to be some of the top students even at schools like MIT. These books are not easy and if you can finish even one, I expect you to have a very strong foundation in mathematics. Note without the answer key, these books are very useless. Don't expect the child to figure out how to solve questions in this book without the answer key (and it is VERY well written). That said, if the child gains a very strong foundation from those books, I really wouldn't worry much about the child's future (students with solid academic background can succeed almost anywhere).
* Study math topics at least 1 semester earlier than when school starts teaching the math topics. Always easier to get familiar with the topic then learn the topic again at school. Great confidence booster and all. I really wish I did this for Chemistry looking back. Thankfully, Khan Academy exists for Chemistry topics.
* Expose to basic coding. Khan Academy is a thing. Kid might like it, kid might not. I think Scratch is very popular to elementary/middle school kids.
* Playing a musical instrument, joining a choir, participating in plays, learning to draw (attending an art school?)
* Going on vacation in different places every summer (National Parks, Singapore, etc.) to expose kids to different cultures/environments

Things I wish I was more exposed to:
* Basic sports. Being healthy/fit is always good (I ended up learning how to skateboard from my elementary school years. Something I doubt kids at privates generally get exposed to)

Don't forget to expose kids to utilize their local libraries. Creating a reading addiction early on can be really beneficial.
Honestly, in my perspective, if the child does the above, regardless of the child studying at a public or private, I would expect the child to have the academics to have a solid chance at schools like MIT.
The contents of AoPS books alone are probably better than the best education one can receive in elementary/middle/high school anywhere in the world for mathematics. Then for Calculus, there's always Spival's Calculus but I highly recommend going through a regular Calculus book first before touching that book (Apostol's Calculus is too dry for my tastes).

At least that's my perspective.
I did note that students in private schools scored much better in standardized tests but I also felt students who didn't do well at private schools also lost a lot of confidence on themselves early on (and were far more depressed throughout the school years). And intellectually, I honestly didn't find much if any difference between top students at a public and top students at a private. If anything, I felt often times the top students at public had more potential. Sure my friends at the private had much higher SAT scores (in 2340s [2370, 35/36 here]) unlike my friends at the public (1900s) but standardized test can be studied for by oneself by just buying some SAT/ACT books and doing the questions (and reviewing each wrong one 3 times by reading through the answer sheet).

As long as the public has the courses necessary for the kid, there's really not much of a difference.
I also felt that being at a public school helped make me more comfortable talking to people of different backgrounds.
I think that soft skill is pretty critical to the 'real world'.

I really don't think there's any elementary/middle/high school education that can replace reading books from the local library + learning to play an instrument + doing some math from really good textbooks with great answer keys + being exposed to robotics and all.
I would personally invest in the child directly for outside school activities/learning than pay the premium for a private school in the early years.
Maybe get a private tutor to help the kid not give up the AoPS books. Those books need some adult (e.g. parents) to keep track of progress and willing to show step by step of the answers in the answer key (it's super well written and honestly, without the answer key, I don't expect any tutor to solve the questions. I wouldn't be surprised myself if I can't solve some of the Algebra 1 questions there today. One thing I wish I can redo is I wish my parents showed me the answer key of the content after I attempted the question. The answer key explanations are far better than what an adult can summarize).

Anyways, at end of day, each student is different so there's no 'right answer'. I just think that regular reading + creating solid foundation in mathematics is all you really need for academics in elementary/middle/high.

---
For the top 20 schools, and particularly the Ivies, they may be no benefit, and attending a top private may be actually be a detriment to admission to one of these types of schools.
If your child attends a private, then your child will need much better academic stats + extracurriculars.
I know a lot of peers at the private school I attended complained due to this. But this is to be expected as those with more privilege should be expected to achieve more academically.
I would argue that it's much easier to attend a top school from a public school than from a private school if the parents ensure the kid is creating a solid foundation reading/writing/math.
There's also some truths to 'birds of the feather, flock together'. If the public is of dreadful quality, then I guess there's not much of an option. However, I feel often times public are of better quality as long as the public has AP courses + allows taking courses with a local college.

I just can't fathom how attending a more expensive elementary school would be of any benefit. Elementary kids don't really learn anything (it's more about developing social skills). Maybe some long division here and there. I really think the 'real education' happens outside school during elementary school years. Parents have to be pretty active in the early years. I don't think throwing money blindly is of much help. Long division is the same in both private or public. The contents with the help of Khan Academy shouldn't need special teachers and all. I honestly don't even think you need 6+ years to learn division/fraction/etc. Even less than a month is enough if the kid is studying by him/herself outside school. I have tutored many elementary kids and I have personally evidenced kids learn 4 years of math within 2~3 months (and that was me just tutoring twice a week). Thus, I really can't see what benefits privates bring in elementary/middle. Worst case scenario, just snatch up some high schooler who is good at math and have him/her tutor the child twice a week for a decent hourly pay + fruits/snacks. Or create study group (e.g.: reading group) by coordinating with the parents who are motivated in the school
Last edited by fwellimort on Wed Jun 16, 2021 1:14 pm, edited 21 times in total.
User avatar
TomatoTomahto
Posts: 17158
Joined: Mon Apr 11, 2011 1:48 pm

Re: Private School - Elementary vs Middle/High (bang for the buck)

Post by TomatoTomahto »

+1 on Art of Problem Solving math textbooks. They were transformative for my son (who didn’t do math competitions, but could have).
I get the FI part but not the RE part of FIRE.
User avatar
leeks
Posts: 1466
Joined: Thu Apr 07, 2011 4:33 pm
Location: virginia

Re: Private School - Elementary vs Middle/High (bang for the buck)

Post by leeks »

TomatoTomahto wrote: Wed Jun 16, 2021 12:43 pm +1 on Art of Problem Solving math textbooks. They were transformative for my son (who didn’t do math competitions, but could have).
My first-grader is using the paper Beast Academy books this year for homeschool and we will stick with them as a supplement even when he goes to a private Montessori next year. Beast Academy is the elementary series by Art of Problem Solving. It can fit into summers/school breaks without displacing too much time for other fun stuff. It's in a fun comic book style.

Since we had an unexpected homeschool year, we have discovered a variety of high quality materials and plan to continue supplementing at home. It is true that academic development should not be only left up to the school, no matter how great the school.

Supplementing a lot at home is how many families with gifted children survive the inadequate public school programs.

Other excellent elementary-age materials:
Building Foundations of Scientific Understanding for science.
The Story of the World for history.
Michael Clay Thompson for language arts.
Royal Fireworks Press publishes a bunch of other high-quality stuff for gifted children (logic, philosophy, latin in age appropriate formats for young kids)
Geotoys for geography puzzles and games (and keep maps, atlases, globe around the house).

Of course the usual sports, music, arts, foreign languages, etc is also valuable. And plenty of time for reading/downtime/play. Tons of books in the house, library trips, and keep reading classic books out loud to children through all elementary years.

A desire for this kind of stuff is part of why we plan to always keep a parent available on weekdays after school so we can supplement our children's education directly and give them plenty of down time at home for independent pursuits.

School is not the only factor in a child's education.
Last edited by leeks on Wed Jun 16, 2021 1:40 pm, edited 2 times in total.
MathWizard
Posts: 6561
Joined: Tue Jul 26, 2011 1:35 pm

Re: Private School - Elementary vs Middle/High (bang for the buck)

Post by MathWizard »

If the reason for choosing private vs. public is academics, I assume that the decision
is because you believe that private provides a superior learning environment.

Most people would go private for K-12 or public for K-12.

If you can only do some, and with the assumption that private is better,
then elementary would be preferable.

Education builds on prior grades.

If you go public then private, your kids will be behind when they go to the private school.

If you go private then public, your kids will be ahead, and can take AP courses so that they do not get bored,
and will have a better chance at top colleges, and may get credit for some general education courses in college.

In public schools, there are lots of elementary schools which feed into 1 or 2 junior highs, then into
a single senor high school. This means that there would already be a transition from K-6 to junior high, or
from K-8 (or K-9) to senior high, so transitioning at those points would mean that you kids would
not be the one kid who is new kid.
Jags4186
Posts: 8198
Joined: Wed Jun 18, 2014 7:12 pm

Re: Private School - Elementary vs Middle/High (bang for the buck)

Post by Jags4186 »

It depends on what you’re sending the kid to private school for. I went to public school through 8th grade. I was sent to a private high school. Where I went to private high school, the “safety” school students matriculated to was my state’s flagship campus. We had like 40 kids go to Rutgers College. We had 15 or so kids out of 180 go to Ivy League schools, several people go to West Point or Annapolis, 20-30 kids go to schools like Boston College, Georgetown, NYU, and Notre Dame.

Looking back on results I, a good but not great student (top 20% of my class, 1400ish SATs) went to the same very highly ranked university my older sister, no.1 in her class high 1500s SATs, president of everything but went to the town public school, went to. Had I been at the public school, I probably would have ended up either a middling university or very expensive private LAC.
halfnine
Posts: 2421
Joined: Tue Dec 21, 2010 12:48 pm

Re: Private School - Elementary vs Middle/High (bang for the buck)

Post by halfnine »

leeks wrote: Tue Jun 15, 2021 5:57 pm
...An anecdote only - we are trying a private Montessori elementary next year for our children entering K and 2nd grades...
In general we wouldn't put our children in a Montessori school that typically allowed for non-Montessori transfer students especially beyond Kindergarten. Not if the reason we are paying is specifically for the Montessori environment. The schools that tend to allow transfers are often more "Montessori" in name than in function. That said, of course, the school could be perfectly fine, a great fit and a great education. I just wouldn't count on it being a true Montessori school.

Disclosure: Our kids were in a Montessori school from ages 3 to 8.
Irisheyes
Posts: 313
Joined: Wed Feb 14, 2018 12:36 pm

Re: Private School - Elementary vs Middle/High (bang for the buck)

Post by Irisheyes »

Our choices were a highly mediocre public school or a fairly good private school. Our daughter wanted to go to the mediocre public school, quite against our wishes, but we honored her choice and she graduated this year in the top 3% of her public school class.

In the end it worked out fine, as she was accepted into top private and public colleges (UCLA, Berkeley, etc) and offered excellent merit scholarships too.

However a ton of parental sweat equity made up for what the public school lacked. Right from Kindergarten when I taught my daughter to read all the way through AP Calc and AP Comp sci, where my partner tutored her on a weekly basis. We made sure she enrolled in meaningful (to admission officers) extra curriculars etc etc. And we spent a ton of time helping her find and apply to suitable colleges, because there was just no help from the school counselors, who are extremely overworked.

It took a ton of our time to make up for the lousy prep she got in public school. So one factor in your equation as you consider public vs private is ohw much time you personally have to help your child along the way.
Wannaretireearly
Posts: 4881
Joined: Wed Mar 31, 2010 4:39 pm

Re: Private School - Elementary vs Middle/High (bang for the buck)

Post by Wannaretireearly »

Irisheyes wrote: Wed Jun 16, 2021 3:49 pm Our choices were a highly mediocre public school or a fairly good private school. Our daughter wanted to go to the mediocre public school, quite against our wishes, but we honored her choice and she graduated this year in the top 3% of her public school class.

In the end it worked out fine, as she was accepted into top private and public colleges (UCLA, Berkeley, etc) and offered excellent merit scholarships too.

However a ton of parental sweat equity made up for what the public school lacked. Right from Kindergarten when I taught my daughter to read all the way through AP Calc and AP Comp sci, where my partner tutored her on a weekly basis. We made sure she enrolled in meaningful (to admission officers) extra curriculars etc etc. And we spent a ton of time helping her find and apply to suitable colleges, because there was just no help from the school counselors, who are extremely overworked.

It took a ton of our time to make up for the lousy prep she got in public school. So one factor in your equation as you consider public vs private is ohw much time you personally have to help your child along the way.
Great points. One reason we're keeping an external college counselor as my elder son moves from private to public at 8th grade this year.
“At some point you are trading time you will never get back for money you will never spend.“ | “How do you want to spend the best remaining year of your life?“
JPM
Posts: 586
Joined: Sun Aug 19, 2018 2:29 pm

Re: Private School - Elementary vs Middle/High (bang for the buck)

Post by JPM »

DS1 attended a cooperative Montessori ages 4&5. Really excellent. learned to read and read well early and still has amazing reading speed. Went to urban magnet school for first and second grade, good experience there with good teachers. Went to religious private school third thru sixth with enrichment school in the summers in kind of a forerunner of the Hopkins programs of today. Seventh and eighth back to public school for a particularly good gifted ed program at that level. Back to religious private school advanced program for high school. Top near-ivy undergrad and never needed grad school. Has had successful careers in banking and government.

DS2 attended Montessori co-op ages 3&4. Similar excellent experience. Learned to read well and also has high reading speed in adult life. Went to urban magnet first grade, religious private second thru sixth and back to public for seventh and eighth for the gifted program. DS2 more social than DS1 and still has a multiethnic group of friends from junior high. Went to religious private high school. Liked his brother's college friends so went to same college and then top ten law school, has own law practice.

DD attended a commercial Montessori that was disappointing. Then for grade school a religious private and being a tomboy and sports kid was the odd girl out among the daintier types so transferred to public school fourth grade. Played sports there, did extracurriculars, and rebelled against academic excellence. At eighth grade she saw that being academic stars had really paid off for her big brothers and talked of turning over a new leaf and began to work at school in eighth grade. By then she was getting a poor education in "average" classes in a second rate exurban public school district. To try to compensate for a poor grade school background, we decided to send her to an expensive (3x the cost of the religious private) prep school where, because of single digit class sizes, she wouldn't be able to hide. If she wasn't getting it, the teachers readily recognized that and saw to it that she learned what she needed to learn. Played three sports there. That was the best school decision we ever made. She made up lost ground, went to flagship public university and works in tech out west.

The Montessori co-op was the best money we ever spent in terms of bang-for-the -buck. Half the price of the commercial Montessori with vastly superior outcomes.

Gifted programs in public schools are under fire from the bien pensant crowd now, alleged to be an engine of unearned privilege so their days may be numbered. DS 1 and DS2 benefited a lot from these programs in urban public schools and so did their multiethnic working class classmates who became educated professionals in adulthood. Maybe the parents will successfully oppose their destruction. If not, many public schools will become wastelands.
rule of law guy
Posts: 525
Joined: Mon Jun 07, 2021 9:35 pm

Re: Private School - Elementary vs Middle/High (bang for the buck)

Post by rule of law guy »

we enrolled our kids in a private school pre-K through high school program, and they profited from the rigor and continuity that provided. one of our kids did opt to go to a great boarding school in high school, which made sense for her but not my son...and she was well prepared for the challenging boarding school because we bit the bullet on private school education from pre-K for her.
long story short, the best gift you can give your kids is your love and the best education that your money can afford
Never wrong, unless my wife tells me that I am.
stoptothink
Posts: 15368
Joined: Fri Dec 31, 2010 8:53 am

Re: Private School - Elementary vs Middle/High (bang for the buck)

Post by stoptothink »

HereToLearn wrote: Wed Jun 16, 2021 12:18 pm
Frank the Tank wrote: Wed Jun 16, 2021 11:03 am There's no one size fits all answer because there are too many variables and it's way too broad to make a "public vs. private" comparison.

Some private schools are highly competitive while others are essentially open admissions as long as you can write a check.

Some public schools are going to be as high quality as any private school while others are going to present issues.

Where we live, the public school districts are very highly rated, so I personally would find it to be a horrible "bang for your buck" to pay for private schools on top of our property taxes (and I outright believe that our children are receiving a better education in our public schools regardless of cost).

If we lived 5 miles away in a different public school district that's much lower rated, the calculus for us would probably be totally different.

With the way college admissions work these days, I personally wouldn't be basing any public/private school decision on any perceived advantage to get into higher-rated colleges. There might be a very small handful of super elite private schools that provide some material advantages (e.g. Philips Exeter, Harvard-Westlake), but a typical upper middle class private high school generally isn't going to have any discernible college admissions advantage over a typical upper middle class public high school. As long as a local public high school provides a full slate of rigorous classes for high performers (e.g. AP classes), it's all about what your kid is able to achieve as opposed to the school itself.

You mentioned that you went to school outside of the US, so please note that elite colleges such as the Ivy League universities focus much more on individual "hooks" since they receive way more applicants with perfect test score/perfect grades than they have spaces for in their classes. Getting into an Ivy League school (or similarly situated ones like Stanford, MIT, etc.) is NOT an objective measure of academic achievement like it is in many (most?) other countries. For instance, 20-25% of each Ivy League class is dedicated to recruited athletes. Frankly, at a typical suburban high school these days (public or private), you're more likely to find a very smart kid that's a superstar at baseball or another sport (e.g. good enough that they'd be getting a Division I athletic scholarship at another university) get into the Ivy League than just a "normal" valedictorian with a perfect academic record.

Another significant percentage is allocated to legacy admissions (children of alumni) - those kids usually have the grades/test scores to get admitted, so they're getting a preference over non-legacy applicants with similar stats. There are also development cases (children of super-high donors). A lot of wealthy high schools may have a larger number of legacy and/or development applicants, so that can skew the numbers (where an applicant really got in because of legacy/development status as opposed to the high school itself). The number of "unhooked" applicants at elite universities is much lower than people think. To be sure, it doesn't mean that it's impossible for a "normal" valedictorian to get into those colleges, but rather it's not a transparent or completely objective process (which is difficult for people that come from countries to understand where college admissions might be based solely on grades and/or test scores) and they're competing for a smaller number of slots compared to recruited athletes (who have virtually guaranteed admissions).

This is a long-winded way of saying that you should find the best educational situation for your children now and don't fool yourself into thinking that one particular school is going to provide a material advantage for getting into colleges compared to another one (assuming that a school isn't clearly deficient, e.g. low-performing, doesn't offer AP classes, etc.).
I agree with all of the above, other than one slight point of clarification. Recruited athletes represent more of the first year class at say Dartmouth than at Penn or Cornell. The Ivies are all subject to the same recruiting agreements--I think it is still called Academic Index for Athletic Recruiting--but my info may be out of date. If I recall correctly, each school recruits 200-250 athletes each year, so a higher percentage at small Dartmouth vs larger Cornell.

Some of the athletic admits are also legacy admits. Fewer development admits than we all think.

Agree with your comment 100%, and will add as long as there is a like-minded peer group: "As long as a local public high school provides a full slate of rigorous classes for high performers (e.g. AP classes), it's all about what your kid is able to achieve as opposed to the school itself."

My children attended public HS but we live in an area with a number of private schools, and a decent # of students who go away to boarding school. Public school grads' outcomes were absolutely comparable.
FWIW there are two Ivy league grads in my family, both recruited athletes (non-money sports, women's water polo and men's soccer). On academic merit alone, neither would have been accepted to our state flagship and they weren't recruited by state flagship for sports either. Very fortunate that they fell right into that perfect spot of good athlete (but not good enough for scholarship at state flagship), good student (but not good enough to get into state flagship on academics alone), right skin color, and poor enough so it was all paid for.
Normchad
Posts: 5648
Joined: Thu Mar 03, 2011 6:20 am

Re: Private School - Elementary vs Middle/High (bang for the buck)

Post by Normchad »

Lots of great advice here. Unfortunately, there is no perfect answer.

We researched the snot out of this question 20 years ago, and decided to send ours to a great private school, starting in kindergarten.

We ended up switching to public schools in first grade. It just wasn’t the right fit for our kid and that school. I still think that particular school is great, and have recommended it to others.

Just a long way of saying there are no “one size fits all” answers.
User avatar
Sandtrap
Posts: 19591
Joined: Sat Nov 26, 2016 5:32 pm
Location: Hawaii No Ka Oi - white sandy beaches, N. Arizona 1 mile high.

Re: Private School - Elementary vs Middle/High (bang for the buck)

Post by Sandtrap »

Actionably:
Highest level private available from k to graduate level with continuity and consistency in all areas including academic, etc.
Results are child dependent.

However………within this one metric of many…..etc…

Financially…..can you pay for continuity??

j🌺
Wiki Bogleheads Wiki: Everything You Need to Know
Arabesque
Posts: 339
Joined: Mon Jan 08, 2018 8:56 am

Re: Private School - Elementary vs Middle/High (bang for the buck)

Post by Arabesque »

It really depends.

Our public was good, not great. Elementary was fine for the basics. My older was driven and smart, and though it was never really the plan, she went to middle and high school at the local college-prep private. It worked brilliantly for her. She was able to learn two languages and lots of calculus, engage in endless extracurriculars and amazing, competitive (free) summer programs. She used every bit of the private and was accepted to an ivy with lots of $$$. Her sister went to the private MS and HS, too. It certainly helped her, but really wasn't necessary.

Alternatively, my dentist sent his daughter to private for middle school and kept her from all that middle school angst. She came back to public high school in good mental shape, well prepared for intense courses. By HS, the public system was willing to track the smart kids and give them lots of attention, APs, opportunities. She did well, and her parents swore by that path.
User avatar
leeks
Posts: 1466
Joined: Thu Apr 07, 2011 4:33 pm
Location: virginia

Re: Private School - Elementary vs Middle/High (bang for the buck)

Post by leeks »

halfnine wrote: Wed Jun 16, 2021 3:16 pm
leeks wrote: Tue Jun 15, 2021 5:57 pm
...An anecdote only - we are trying a private Montessori elementary next year for our children entering K and 2nd grades...
In general we wouldn't put our children in a Montessori school that typically allowed for non-Montessori transfer students especially beyond Kindergarten. Not if the reason we are paying is specifically for the Montessori environment. The schools that tend to allow transfers are often more "Montessori" in name than in function. That said, of course, the school could be perfectly fine, a great fit and a great education. I just wouldn't count on it being a true Montessori school.
I hear your concern. They are an accredited, well-established Montessori with properly trained teachers and such (a nonprofit overseen by a Board of Trustees). It is a relatively large school (up to 8th grade) and they do have more concern for achieving "challenging" academics than some Montessori programs (part of the appeal for us). They have three lower elementary (mixed grade 1-3) classrooms so there is always a bit of turnover (people move, can't afford it past a certain grade, etc) and they do fill empty spaces to keep roughly the same number of kids per grade.

There are fewer upper elementary and middle school classes though. I think a lot of parents find the most value in the early years but are willing to save money and switch to public as kids get older and the public offerings are more differentiated (there are upper elementary public gifted and specialty middle schools in the region). Although the pandemic changed that for some parents and there was talk of adding a third upper elementary class as more families wanted to stay next year.

But yeah, we had to prove our bona fides. They interview children and parents to determine academic levels and fit with Montessori style (they consistently have a waitlist so they get their pick of new students).

My mother was a Montessori teacher (I attended for preschool), I have been mostly a stay at home parent and our home life is reasonably consistent with Montessori values. My kids are self-driven, curious and (fiercely) independent. They can focus on a single task for a long time and they enjoy completing challenging things. We didn't have access to a Montessori preschool (they did half-day Reggio Emilia) but I feel they are well-suited to Montessori methods (and I realize that there will still be an adjustment phase).

What I can't know in advance is whether the school will live up to the reputation/promises and that the kids will be compatible with their specific teachers. That is a gamble as it would be with any school. We have our fingers crossed but I am willing to pull them out to temporarily homeschool again if it isn't as good a fit as we hope it will be.
halfnine wrote: Wed Jun 16, 2021 3:16 pm Disclosure: Our kids were in a Montessori school from ages 3 to 8.
Did your kids attend public schools after age 8?
Last edited by leeks on Wed Jun 16, 2021 9:21 pm, edited 1 time in total.
HereToLearn
Posts: 1537
Joined: Sat Mar 17, 2018 5:53 pm

Re: Private School - Elementary vs Middle/High (bang for the buck)

Post by HereToLearn »

Arabesque wrote: Wed Jun 16, 2021 7:50 pm It really depends.

Our public was good, not great. Elementary was fine for the basics. My older was driven and smart, and though it was never really the plan, she went to middle and high school at the local college-prep private. It worked brilliantly for her. She was able to learn two languages and lots of calculus, engage in endless extracurriculars and amazing, competitive (free) summer programs. She used every bit of the private and was accepted to an ivy with lots of $$$. Her sister went to the private MS and HS, too. It certainly helped her, but really wasn't necessary.

Alternatively, my dentist sent his daughter to private for middle school and kept her from all that middle school angst. She came back to public high school in good mental shape, well prepared for intense courses. By HS, the public system was willing to track the smart kids and give them lots of attention, APs, opportunities. She did well, and her parents swore by that path.
When you say accepted to an Ivy with lots of $$$, do you mean the school had a large endowment? The Ivies do not offer merit awards other than some very small dollar research grant type of awards (a few thousand a year perhaps). If an Ivy reduced the cost of tuition, that was financial aid.
HereToLearn
Posts: 1537
Joined: Sat Mar 17, 2018 5:53 pm

Re: Private School - Elementary vs Middle/High (bang for the buck)

Post by HereToLearn »

fwellimort wrote: Wed Jun 16, 2021 12:37 pm I attended both public and private during my pre-college academic years.
I think I ended up pretty well academically considering I ended up graduating from Columbia Univ in City of NY.

From my experience, there's not much of a difference between a decent public and a good private school.
Sure the good private school has better academics and all but I felt the differences were pretty minute and not worth the additional costs.

What I felt were of the greatest help academically were:
* Having my parents expose me to lego robotics (exposes to basic engineering and programming) through FIRST Robotics and all
* Building habit of reading books regularly (Harry Potter, Twilight, Eragon, Artemis Fowl, Twilight, Percy Jackson, etc.)
* Building strong foundation of mathematics through doing lots of arithmetic early on and then doing the Art of Problem Solving math textbooks (these are VERY difficult and I wouldn't expect even college graduates to do all the questions from even Pre-Algebra have they never been exposed to competition math).
AoPS books are THE math books to do if you want a really solid foundation in math. In my mind, the best math books. If you can do all the AoPS books, then I expect you to be some of the top students even at schools like MIT. These books are not easy and if you can finish even one, I expect you to have a very strong foundation in mathematics. Note without the answer key, these books are very useless. Don't expect the child to figure out how to solve questions in this book without the answer key (and it is VERY well written). That said, if the child gains a very strong foundation from those books, I really wouldn't worry much about the child's future (students with solid academic background can succeed almost anywhere).
* Study math topics at least 1 semester earlier than when school starts teaching the math topics. Always easier to get familiar with the topic then learn the topic again at school. Great confidence booster and all. I really wish I did this for Chemistry looking back. Thankfully, Khan Academy exists for Chemistry topics.
* Expose to basic coding. Khan Academy is a thing. Kid might like it, kid might not. I think Scratch is very popular to elementary/middle school kids.
* Playing a musical instrument, joining a choir, participating in plays, learning to draw (attending an art school?)
* Going on vacation in different places every summer (National Parks, Singapore, etc.) to expose kids to different cultures/environments

Things I wish I was more exposed to:
* Basic sports. Being healthy/fit is always good (I ended up learning how to skateboard from my elementary school years. Something I doubt kids at privates generally get exposed to)

Don't forget to expose kids to utilize their local libraries. Creating a reading addiction early on can be really beneficial.
Honestly, in my perspective, if the child does the above, regardless of the child studying at a public or private, I would expect the child to have the academics to have a solid chance at schools like MIT.
The contents of AoPS books alone are probably better than the best education one can receive in elementary/middle/high school anywhere in the world for mathematics. Then for Calculus, there's always Spival's Calculus but I highly recommend going through a regular Calculus book first before touching that book (Apostol's Calculus is too dry for my tastes).

At least that's my perspective.
I did note that students in private schools scored much better in standardized tests but I also felt students who didn't do well at private schools also lost a lot of confidence on themselves early on (and were far more depressed throughout the school years). And intellectually, I honestly didn't find much if any difference between top students at a public and top students at a private. If anything, I felt often times the top students at public had more potential. Sure my friends at the private had much higher SAT scores (in 2340s [2370, 35/36 here]) unlike my friends at the public (1900s) but standardized test can be studied for by oneself by just buying some SAT/ACT books and doing the questions (and reviewing each wrong one 3 times by reading through the answer sheet).

As long as the public has the courses necessary for the kid, there's really not much of a difference.
I also felt that being at a public school helped make me more comfortable talking to people of different backgrounds.
I think that soft skill is pretty critical to the 'real world'.

I really don't think there's any elementary/middle/high school education that can replace reading books from the local library + learning to play an instrument + doing some math from really good textbooks with great answer keys + being exposed to robotics and all.
I would personally invest in the child directly for outside school activities/learning than pay the premium for a private school in the early years.
Maybe get a private tutor to help the kid not give up the AoPS books. Those books need some adult (e.g. parents) to keep track of progress and willing to show step by step of the answers in the answer key (it's super well written and honestly, without the answer key, I don't expect any tutor to solve the questions. I wouldn't be surprised myself if I can't solve some of the Algebra 1 questions there today. One thing I wish I can redo is I wish my parents showed me the answer key of the content after I attempted the question. The answer key explanations are far better than what an adult can summarize).

Anyways, at end of day, each student is different so there's no 'right answer'. I just think that regular reading + creating solid foundation in mathematics is all you really need for academics in elementary/middle/high.

---
For the top 20 schools, and particularly the Ivies, they may be no benefit, and attending a top private may be actually be a detriment to admission to one of these types of schools.
If your child attends a private, then your child will need much better academic stats + extracurriculars.
I know a lot of peers at the private school I attended complained due to this. But this is to be expected as those with more privilege should be expected to achieve more academically.
I would argue that it's much easier to attend a top school from a public school than from a private school if the parents ensure the kid is creating a solid foundation reading/writing/math.
There's also some truths to 'birds of the feather, flock together'. If the public is of dreadful quality, then I guess there's not much of an option. However, I feel often times public are of better quality as long as the public has AP courses + allows taking courses with a local college.

I just can't fathom how attending a more expensive elementary school would be of any benefit. Elementary kids don't really learn anything (it's more about developing social skills). Maybe some long division here and there. I really think the 'real education' happens outside school during elementary school years. Parents have to be pretty active in the early years. I don't think throwing money blindly is of much help. Long division is the same in both private or public. The contents with the help of Khan Academy shouldn't need special teachers and all. I honestly don't even think you need 6+ years to learn division/fraction/etc. Even less than a month is enough if the kid is studying by him/herself outside school. I have tutored many elementary kids and I have personally evidenced kids learn 4 years of math within 2~3 months (and that was me just tutoring twice a week). Thus, I really can't see what benefits privates bring in elementary/middle. Worst case scenario, just snatch up some high schooler who is good at math and have him/her tutor the child twice a week for a decent hourly pay + fruits/snacks. Or create study group (e.g.: reading group) by coordinating with the parents who are motivated in the school
I agree with so much of what you have written above, but I am in awe of your parents intentionally exposing you to so many outside academics, and then following through to see that you completed the work. It never occurred to me to do more than read to my children from birth, and support their interests, if they expressed any. I did track down one or two programs (Splash and an online summer math class at CTY in order to skip a year of math), but otherwise, I followed their leads.

They discovered Scratch because a local elementary school teacher was passionate about it, so they both dove into it.

The Scratch developers have since created a program for K-2, a sort of pre-Scratch, to introduce the idea of coding to even younger children. I don't know when this was released as my children outgrew Scratch, but they enjoyed it for a while. My now 22 year old was so enamored of Scratch when he was eight or nine that he contacted the developers to suggest new 'blocks' he wanted them to create. The highlight of a summer trip to Boston was an inside tour of the MIT Media Lab.

My son found AoPS by himself and sent me the links to order the books when he was in middle school. I don't know what he did with them, but I know he has recommended them to others.

I do think a child's peer group is vitally important, and we were fortunate to live in a school district which offered challenging courses, but most importantly, a group of peers interested in pursuing academic enrichment outside of school.

I agree with NormChad who wrote that while there is lots of great advice here, there is no perfect answer.
oldfort
Posts: 2781
Joined: Mon Mar 02, 2020 7:45 pm

Re: Private School - Elementary vs Middle/High (bang for the buck)

Post by oldfort »

sdsu04 wrote: Tue Jun 15, 2021 5:20 pm 4. Do private schools prepare the child and provide a better opportunity to a higher ranked college (or a college of their choice)? Understand this is not a yes/no answer but your experience would be helpful to learn about and help us better understand their value.
There's an admissions advantage to attending a feeder school for admissions to the Ivies. 5% of Harvard freshman came from one of 7 schools. Feeder schools are usually private, but some, such as Stuyvesant, are public magnets.
https://www.thecrimson.com/article/2013 ... r-schools/
oldfort
Posts: 2781
Joined: Mon Mar 02, 2020 7:45 pm

Re: Private School - Elementary vs Middle/High (bang for the buck)

Post by oldfort »

sdsu04 wrote: Tue Jun 15, 2021 5:20 pm 1. Is it better to invest In a Private school education during elementary school years - which are formative years for the child?
This is a clear no to me. When applying to colleges, college see your high school name, your high school transcript, your high school LORs, and your high school ECs. At least to the extent you care about college admissions: high school >>> middle school.
qwertyjazz
Posts: 2000
Joined: Tue Feb 23, 2016 3:24 am

Re: Private School - Elementary vs Middle/High (bang for the buck)

Post by qwertyjazz »

oldfort wrote: Wed Jun 16, 2021 10:04 pm
sdsu04 wrote: Tue Jun 15, 2021 5:20 pm 4. Do private schools prepare the child and provide a better opportunity to a higher ranked college (or a college of their choice)? Understand this is not a yes/no answer but your experience would be helpful to learn about and help us better understand their value.
There's an admissions advantage to attending a feeder school for admissions to the Ivies. 5% of Harvard freshman came from one of 7 schools. Feeder schools are usually private, but some, such as Stuyvesant, are public magnets.
https://www.thecrimson.com/article/2013 ... r-schools/
But the underlying question is whether you would get into a feeder school in the first place without excellent elementary school. The link you put described admission rates of the feeder schools as pretty low. The argument is basically you are just moving up the selection process to high school from college. Similar argument is college does not matter given those who would get into a good school are those selected to succeed regardless. That all comes from being in the correct preprimary class. Your entire future is determined by how you do at 3-4 years of age. The rest is just selection based on that :wink:
G.E. Box "All models are wrong, but some are useful."
User avatar
leeks
Posts: 1466
Joined: Thu Apr 07, 2011 4:33 pm
Location: virginia

Re: Private School - Elementary vs Middle/High (bang for the buck)

Post by leeks »

qwertyjazz wrote: Wed Jun 16, 2021 11:32 pm
oldfort wrote: Wed Jun 16, 2021 10:04 pm
sdsu04 wrote: Tue Jun 15, 2021 5:20 pm 4. Do private schools prepare the child and provide a better opportunity to a higher ranked college (or a college of their choice)? Understand this is not a yes/no answer but your experience would be helpful to learn about and help us better understand their value.
There's an admissions advantage to attending a feeder school for admissions to the Ivies. 5% of Harvard freshman came from one of 7 schools. Feeder schools are usually private, but some, such as Stuyvesant, are public magnets.
https://www.thecrimson.com/article/2013 ... r-schools/
But the underlying question is whether you would get into a feeder school in the first place without excellent elementary school. The link you put described admission rates of the feeder schools as pretty low. The argument is basically you are just moving up the selection process to high school from college. Similar argument is college does not matter given those who would get into a good school are those selected to succeed regardless. That all comes from being in the correct preprimary class. Your entire future is determined by how you do at 3-4 years of age. The rest is just selection based on that :wink:
In which case one can suggest high value for investing in ages 0-4 by having a stay at home parent (who is highly educated and reads up on early childhood development) combined with a half-day (fully accredited) Montessori program for ages 2-4. We didn't have Montessori access at those ages (used half-day Reggio Emilia) but I think it is the best of the preschool approaches (for most kids, when done well).

The idea that "Your entire future is determined by how you do at 3-4 years of age." is not all :wink: - it is one justification for publicly funding preK and there are some studies that find lifetime benefits from attending preK.

However caregiving/preschool is arranged, I don't think it is possible to overstate the need to invest in the early childhood years while the brain is developing by providing adequate nurture, security, nutrition, stimulation, movement, language exposure, reading, music, etc. I would encourage parents to prioritize those years highest of all.
qwertyjazz
Posts: 2000
Joined: Tue Feb 23, 2016 3:24 am

Re: Private School - Elementary vs Middle/High (bang for the buck)

Post by qwertyjazz »

leeks wrote: Wed Jun 16, 2021 11:49 pm
qwertyjazz wrote: Wed Jun 16, 2021 11:32 pm
oldfort wrote: Wed Jun 16, 2021 10:04 pm
sdsu04 wrote: Tue Jun 15, 2021 5:20 pm 4. Do private schools prepare the child and provide a better opportunity to a higher ranked college (or a college of their choice)? Understand this is not a yes/no answer but your experience would be helpful to learn about and help us better understand their value.
There's an admissions advantage to attending a feeder school for admissions to the Ivies. 5% of Harvard freshman came from one of 7 schools. Feeder schools are usually private, but some, such as Stuyvesant, are public magnets.
https://www.thecrimson.com/article/2013 ... r-schools/
But the underlying question is whether you would get into a feeder school in the first place without excellent elementary school. The link you put described admission rates of the feeder schools as pretty low. The argument is basically you are just moving up the selection process to high school from college. Similar argument is college does not matter given those who would get into a good school are those selected to succeed regardless. That all comes from being in the correct preprimary class. Your entire future is determined by how you do at 3-4 years of age. The rest is just selection based on that :wink:
In which case one can suggest high value for investing in ages 0-4 by having a stay at home parent (who is highly educated and reads up on early childhood development) combined with a half-day (fully accredited) Montessori program for ages 2-4. We didn't have Montessori access at those ages (used half-day Reggio Emilia) but I think it is the best of the preschool approaches (for most kids, when done well).

The idea that "Your entire future is determined by how you do at 3-4 years of age." is not all :wink: - it is one justification for publicly funding preK and there are some studies that find lifetime benefits from attending preK.
The question with headstart data is how much it was due to children improvement vs allowing parents employment.

https://www.google.com/amp/s/www.vox.co ... ds-fund-it
G.E. Box "All models are wrong, but some are useful."
User avatar
leeks
Posts: 1466
Joined: Thu Apr 07, 2011 4:33 pm
Location: virginia

Re: Private School - Elementary vs Middle/High (bang for the buck)

Post by leeks »

qwertyjazz wrote: Wed Jun 16, 2021 11:58 pm
The question with headstart data is how much it was due to children improvement vs allowing parents employment.

https://www.google.com/amp/s/www.vox.co ... ds-fund-it
However the benefits accrue, they appear to be long-lasting:
https://www.npr.org/sections/money/2021 ... t-stronger

I'm not attempting to make the case for a particular public policy.

I am simply acknowledging that experiences at ages 0-4 are indeed formative. Parents who wish to set their children on the best possible educational path should be optimizing the early childhood years first (and there are a range of opinions on what is optimal).
Arabesque
Posts: 339
Joined: Mon Jan 08, 2018 8:56 am

Re: Private School - Elementary vs Middle/High (bang for the buck)

Post by Arabesque »

HereToLearn wrote: Wed Jun 16, 2021 9:17 pm
Arabesque wrote: Wed Jun 16, 2021 7:50 pm It really depends.

Our public was good, not great. Elementary was fine for the basics. My older was driven and smart, and though it was never really the plan, she went to middle and high school at the local college-prep private. It worked brilliantly for her. She was able to learn two languages and lots of calculus, engage in endless extracurriculars and amazing, competitive (free) summer programs. She used every bit of the private and was accepted to an ivy with lots of $$$. Her sister went to the private MS and HS, too. It certainly helped her, but really wasn't necessary..
When you say accepted to an Ivy with lots of $$$, do you mean the school had a large endowment? The Ivies do not offer merit awards other than some very small dollar research grant type of awards (a few thousand a year perhaps). If an Ivy reduced the cost of tuition, that was financial aid.
Yes, obviously it was financial aid. I did not write merit. Harvard is free for families who make less than $80000 and about 10% of income for families with income from 80000-150000. Private school and college cost about the same as a standard state college education for my elder daughter.
learnfpga
Posts: 104
Joined: Mon Jan 25, 2010 3:42 pm

Re: Private School - Elementary vs Middle/High (bang for the buck)

Post by learnfpga »

fwellimort wrote: Wed Jun 16, 2021 12:37 pm I attended both public and private during my pre-college academic years.
I think I ended up pretty well academically considering I ended up graduating from Columbia Univ in City of NY.

From my experience, there's not much of a difference between a decent public and a good private school.
Sure the good private school has better academics and all but I felt the differences were pretty minute and not worth the additional costs.

What I felt were of the greatest help academically were:
* Having my parents expose me to lego robotics (exposes to basic engineering and programming) through FIRST Robotics and all
* Building habit of reading books regularly (Harry Potter, Twilight, Eragon, Artemis Fowl, Twilight, Percy Jackson, etc.)
* Building strong foundation of mathematics through doing lots of arithmetic early on and then doing the Art of Problem Solving math textbooks (these are VERY difficult and I wouldn't expect even college graduates to do all the questions from even Pre-Algebra have they never been exposed to competition math).
AoPS books are THE math books to do if you want a really solid foundation in math. In my mind, the best math books. If you can do all the AoPS books, then I expect you to be some of the top students even at schools like MIT. These books are not easy and if you can finish even one, I expect you to have a very strong foundation in mathematics. Note without the answer key, these books are very useless. Don't expect the child to figure out how to solve questions in this book without the answer key (and it is VERY well written). That said, if the child gains a very strong foundation from those books, I really wouldn't worry much about the child's future (students with solid academic background can succeed almost anywhere).
* Study math topics at least 1 semester earlier than when school starts teaching the math topics. Always easier to get familiar with the topic then learn the topic again at school. Great confidence booster and all. I really wish I did this for Chemistry looking back. Thankfully, Khan Academy exists for Chemistry topics.
* Expose to basic coding. Khan Academy is a thing. Kid might like it, kid might not. I think Scratch is very popular to elementary/middle school kids.
* Playing a musical instrument, joining a choir, participating in plays, learning to draw (attending an art school?)
* Going on vacation in different places every summer (National Parks, Singapore, etc.) to expose kids to different cultures/environments

Things I wish I was more exposed to:
* Basic sports. Being healthy/fit is always good (I ended up learning how to skateboard from my elementary school years. Something I doubt kids at privates generally get exposed to)

Don't forget to expose kids to utilize their local libraries. Creating a reading addiction early on can be really beneficial.
Honestly, in my perspective, if the child does the above, regardless of the child studying at a public or private, I would expect the child to have the academics to have a solid chance at schools like MIT.
The contents of AoPS books alone are probably better than the best education one can receive in elementary/middle/high school anywhere in the world for mathematics. Then for Calculus, there's always Spival's Calculus but I highly recommend going through a regular Calculus book first before touching that book (Apostol's Calculus is too dry for my tastes).

At least that's my perspective.
I did note that students in private schools scored much better in standardized tests but I also felt students who didn't do well at private schools also lost a lot of confidence on themselves early on (and were far more depressed throughout the school years). And intellectually, I honestly didn't find much if any difference between top students at a public and top students at a private. If anything, I felt often times the top students at public had more potential. Sure my friends at the private had much higher SAT scores (in 2340s [2370, 35/36 here]) unlike my friends at the public (1900s) but standardized test can be studied for by oneself by just buying some SAT/ACT books and doing the questions (and reviewing each wrong one 3 times by reading through the answer sheet).

As long as the public has the courses necessary for the kid, there's really not much of a difference.
I also felt that being at a public school helped make me more comfortable talking to people of different backgrounds.
I think that soft skill is pretty critical to the 'real world'.

I really don't think there's any elementary/middle/high school education that can replace reading books from the local library + learning to play an instrument + doing some math from really good textbooks with great answer keys + being exposed to robotics and all.
I would personally invest in the child directly for outside school activities/learning than pay the premium for a private school in the early years.
Maybe get a private tutor to help the kid not give up the AoPS books. Those books need some adult (e.g. parents) to keep track of progress and willing to show step by step of the answers in the answer key (it's super well written and honestly, without the answer key, I don't expect any tutor to solve the questions. I wouldn't be surprised myself if I can't solve some of the Algebra 1 questions there today. One thing I wish I can redo is I wish my parents showed me the answer key of the content after I attempted the question. The answer key explanations are far better than what an adult can summarize).

Anyways, at end of day, each student is different so there's no 'right answer'. I just think that regular reading + creating solid foundation in mathematics is all you really need for academics in elementary/middle/high.

---
For the top 20 schools, and particularly the Ivies, they may be no benefit, and attending a top private may be actually be a detriment to admission to one of these types of schools.
If your child attends a private, then your child will need much better academic stats + extracurriculars.
I know a lot of peers at the private school I attended complained due to this. But this is to be expected as those with more privilege should be expected to achieve more academically.
I would argue that it's much easier to attend a top school from a public school than from a private school if the parents ensure the kid is creating a solid foundation reading/writing/math.
There's also some truths to 'birds of the feather, flock together'. If the public is of dreadful quality, then I guess there's not much of an option. However, I feel often times public are of better quality as long as the public has AP courses + allows taking courses with a local college.

I just can't fathom how attending a more expensive elementary school would be of any benefit. Elementary kids don't really learn anything (it's more about developing social skills). Maybe some long division here and there. I really think the 'real education' happens outside school during elementary school years. Parents have to be pretty active in the early years. I don't think throwing money blindly is of much help. Long division is the same in both private or public. The contents with the help of Khan Academy shouldn't need special teachers and all. I honestly don't even think you need 6+ years to learn division/fraction/etc. Even less than a month is enough if the kid is studying by him/herself outside school. I have tutored many elementary kids and I have personally evidenced kids learn 4 years of math within 2~3 months (and that was me just tutoring twice a week). Thus, I really can't see what benefits privates bring in elementary/middle. Worst case scenario, just snatch up some high schooler who is good at math and have him/her tutor the child twice a week for a decent hourly pay + fruits/snacks. Or create study group (e.g.: reading group) by coordinating with the parents who are motivated in the school
Hi, Thank you for sharing your personal experience. It is very helpful and aligns with my belief as we raise our children. Similar to you we have exposed them to reading, AoPS, Khan, Singapore Math, Scratch/Lego robotics, sports, musical instrument etc. And thankfully they have responded well to most of those things. I hope to travel more and agree that they need more exposure to people, places from different cultures and backgrounds.

What I would like to know and was not addressed in your post is the network effect of private schools. Do you think network of a selective private school is more beneficial in later life compared to a good public school? How big of a an affect does a network have on future success and opportunities.
kj10
Posts: 34
Joined: Mon Jul 06, 2020 11:01 am

Re: Private School - Elementary vs Middle/High (bang for the buck)

Post by kj10 »

fwellimort wrote: Wed Jun 16, 2021 12:37 pm I just can't fathom how attending a more expensive elementary school would be of any benefit. Elementary kids don't really learn anything (it's more about developing social skills). Maybe some long division here and there. I really think the 'real education' happens outside school during elementary school years. Parents have to be pretty active in the early years. I don't think throwing money blindly is of much help. Long division is the same in both private or public. The contents with the help of Khan Academy shouldn't need special teachers and all. I honestly don't even think you need 6+ years to learn division/fraction/etc. Even less than a month is enough if the kid is studying by him/herself outside school. I have tutored many elementary kids and I have personally evidenced kids learn 4 years of math within 2~3 months (and that was me just tutoring twice a week). Thus, I really can't see what benefits privates bring in elementary/middle. Worst case scenario, just snatch up some high schooler who is good at math and have him/her tutor the child twice a week for a decent hourly pay + fruits/snacks. Or create study group (e.g.: reading group) by coordinating with the parents who are motivated in the school
I firmly disagree with this quoted part about “elementary kids don’t really learn anything...” Developmentally, research shows that 0-6 is the most important age range for children. Hence, preschool and early elementary are crucial periods in a child’s academic development. They are not only learning peer-to-peer social skills as you mention, but also what school is and being oriented to perspectives on learning. The pedagogical approach matters immensely. Is school portrayed to the student as a “sit, listen, and do as told” (follow orders + wait to be shown something...the traditional American public school) or is school portrayed as “explore and challenge - I’ll be here to help you as you go” (be independent, seek out problems that interest you, seek peer or teacher help as needed). Another example would be this two different schools: School 1 has speakers and teachers telling students “You are the future! We can’t wait to see the changes you make in the world one day!” While this is cute and inspirational, it is marginalizing. Kids in schools aren’t the future; they are the present! They have real ideas, real feelings, and real abilities. They need not wait to be adults to use those skills and talents. School 2 has speakers and teachers that tell kids - “What do see around you in the world that needs to change? How can you change them and what do you need to do to be successful at that?” I know these sound like subtle differences, but they are radically different valuations of children’s abilities, feelings, and needs- which again isn’t explicitly “taught” but is learned through schooling. These orientations matter so much for children developmentally and they are being formed during the earliest schooling periods for children. Granted, these are reinforced or pushed against over the child’s entire time in school, but the earlier periods are the most formative. There’s a lot of fascinating child development, social psych, sociology of schooling (especially work on the hidden curriculum of schooling), and educational literature that examines this.

Aside from how pedagogical approaches in elementary matter, I will just say that elementary children can really learn a lot, a lot more than just social skills. Maria Montessori’s work covers this explicitly and I encourage anyone unfamiliar with Montessori’s work or a true Montessori program to check out Montessori’s writings and pedagogical approach. Long division is taught differently in a Montessori program, with different student outcomes that the traditional school system model because the focus on the concrete to abstract in math in the Montessori classroom differs greatly from the traditional American public school.

So my comment to the OP would be - select the school that is the best educational fit for your children. It might be public, it might be private. There are fabulous public schools and horrible private schools. Research doesn’t support one over the other because private does not always equal better. Pick the school where your children will be encouraged to flourish developmentally (in all ways) and be ready to re-evaluate each year (or at the very least at each academic stage) if that school is the right one for your children at that point in their lives.
User avatar
leeks
Posts: 1466
Joined: Thu Apr 07, 2011 4:33 pm
Location: virginia

Re: Private School - Elementary vs Middle/High (bang for the buck)

Post by leeks »

:happy
kj10 wrote: Thu Jun 17, 2021 8:23 am
fwellimort wrote: Wed Jun 16, 2021 12:37 pm I just can't fathom how attending a more expensive elementary school would be of any benefit. Elementary kids don't really learn anything (it's more about developing social skills). Maybe some long division here and there. I really think the 'real education' happens outside school during elementary school years. ...
I firmly disagree with this quoted part about “elementary kids don’t really learn anything...” Developmentally, research shows that 0-6 is the most important age range for children. Hence, preschool and early elementary are crucial periods in a child’s academic development.
I think you are both right. Most US public school kids seem to be learning close to nothing in their elementary schools. And a surprising number of parents are willing to tolerate that. Of course supportive parents and a rich home life matters. But kids *could and should* learn stuff in elementary school.

This country is just failing at curriculum - even when teachers are decent and the school has adequate resources and families who care about education. I used to think poor education was only due to poverty and inadequate funding. Those are real factors in some places, but I have seen that the curriculum and instructional approaches are inadequate.

Looking back on my own education, I didn't feel I learned much in the classroom until middle school (even with a grade skip and a once-a-week gifted program). Most of my education at that point was from Montessori preschool, my family, and the library. I thought that was just what elementary age means.

Then my oldest started K and I began questioning the curriculum and how extensively we would need to supplement at home to keep our kids intellectually stimulated. Pandemic led to homeschool for first grade.

I read "The Well Trained Mind" by Susan Bauer (classical education approach) and books on other homeschooling approaches and explored curriculum options and recommended book lists. Now I realize that elementary children can get a rigorous and broad education filled with a variety of subjects in a way that can be fun, adapted to the child's abilities, and does not have to feel like "busy work" or have a huge emphasis on tests. And it can build executive functioning skills and lay a proper foundation for higher study.

And now we are willing to pay for a private school if they can provide that (we will give it a try). I don't think all private schools are better than all public but I hope we find the right fit.

I have a master's degree and always top test scores, etc. But in doing homeschool with my son, I have realized how many "holes" my own public education had. I have learned a ton from an elementary science curriculum even though I had AP bio, AP physics and college chemistry. Never really having been challenged, I did not have to develop a work ethic or adequate executive functioning skills. I did not really learn how to study (I could cram to ace a test but did not retain the knowledge later). I did not get a grounding in philosophy, history before US founding, literature, poetry, and I thought I just disliked those subjects. I can write flawlessly if needed (social media posts not being an example) but I dislike it and do not have a good approach to organizing writing tasks. I know when grammar and spelling are incorrect but I didn't learn any of the terminology and patterns of grammar/spelling until I was doing it with my son. I did not learn geography except as an adult who loves to travel. I did not have exposure to theoretical or conceptual math - I could solve any problem but failed to see the context or beauty in math (my husband is a math geek and I don't have the background to understand when he talks about it). I never had enough geometry. I absorbed up tons of facts from being an addicted reader - but it was never pulled together into organized, cohesive knowledge.

My high school options were better (I had plenty of APs, Latin and French) and I had access to extra enrichment stuff too. But my elementary years were wasted and I think I could have benefitted from a more rigorous foundation. I would have been better prepared for college and life. The public elementary options now are even worse.

I would challenge public elementary parents to get a copy of E. D. Hirsch's book for their child's grade - "What your Nth grader needs to know" - a sort of cliff notes format - and see how much of that content (or any other content) is actually happening in your child's school.

"Rethinking School" is another book by Susan Bauer that I recommend to any parent. It covers a wide range of approaches to ensure a child is well educated whether the child stays in public school or not.
MMiroir
Posts: 852
Joined: Sun Mar 29, 2020 12:14 pm

Re: Private School - Elementary vs Middle/High (bang for the buck)

Post by MMiroir »

oldfort wrote: Wed Jun 16, 2021 10:04 pm
sdsu04 wrote: Tue Jun 15, 2021 5:20 pm 4. Do private schools prepare the child and provide a better opportunity to a higher ranked college (or a college of their choice)? Understand this is not a yes/no answer but your experience would be helpful to learn about and help us better understand their value.
There's an admissions advantage to attending a feeder school for admissions to the Ivies. 5% of Harvard freshman came from one of 7 schools. Feeder schools are usually private, but some, such as Stuyvesant, are public magnets.
https://www.thecrimson.com/article/2013 ... r-schools/
Many of those feeder schools remain feeders because alumni send their kids there. My daughter is at an Ivy, and knows six girls who were accepted from the same small all-girls schools in Connecticut. All were legacies. So the school is a great feeder if you are a legacy, not so much if you are not.
kaliwi
Posts: 10
Joined: Thu Jun 17, 2021 9:54 am

Re: Private School - Elementary vs Middle/High (bang for the buck)

Post by kaliwi »

I would say to take a close look at standardized test scores in the school districts that you're thinking about moving to. This information can typically be found on the state govermnments website. A lot of the third party sites that assign grades to K-12 schools do so on arbitrary measures or things that may not be important to you.

Don't just assume that the private schools perform better. You'll also want to contact the private schools that you're looking at and request grade level summarized testing data. Some publish this info online, but not all.

I personally live in a school district that consistently tests in the bottom 20% of the state. Private clearly outperforms academically and the students seem siginificantly more respectful and well adjusted. I would be happy to send my kids to a public school that at least meets most of the basic state testing benchmarks (and 529 all that money), but here that means buying 2-3x the house.

Something tells me that private schooling in the area that you're looking at might be insanely expensive. Add private schooling for each kid to the cost of a cheaper home. Is it still significantly cheaper than a more expensive home in a good or even decent school district?

To answer one of the original questions, if you're set on private school, but are debating early elementary vs middle school.. I feel like there is greater value at an early elementary age. Kids need to start school off on the right foot, and in my experience, private schools seem to provide a better enviornment for that- typically smaller class sizes and higher expectations.
Last edited by kaliwi on Thu Jun 17, 2021 11:15 am, edited 1 time in total.
MaxDakota
Posts: 30
Joined: Fri Apr 23, 2021 11:21 am

Re: Private School - Elementary vs Middle/High (bang for the buck)

Post by MaxDakota »

If you measure bang for your buck as college admissions, then 9-12th at a top feeder private school delivers. At the school I went to, if you were in the top 25% of the class, you always had multiple Ivies to pick from. The state flagship honors program and various highly ranked liberal arts schools were our safety schools, but again, only if you were at the top of the class. Compared to prep school, college was easy.

To your point though, it is a lot harder to get into such a private school in later grades - over half our graduating class started at this school at some point in elementary school, and if memory serves, we had only 1-2 new kids each in 9th and 10th, with no spots at all for 11th and 12th. So as PPs have said, if you can afford it, private K-12 is ideal.

Our local public schools were excellent, and also sent their fair share of kids to the Ivies. I think most parents, including mine, chose private school anyway for the rigor of the academics, the amazing extracurricular opportunities, and the networking opportunities.
fwellimort
Posts: 890
Joined: Tue Feb 12, 2019 8:41 am

Re: Private School - Elementary vs Middle/High (bang for the buck)

Post by fwellimort »

kj10 wrote: Thu Jun 17, 2021 8:23 am I firmly disagree with this quoted part about “elementary kids don’t really learn anything...” Developmentally, research shows that 0-6 is the most important age range for children. Hence, preschool and early elementary are crucial periods in a child’s academic development. They are not only learning peer-to-peer social skills as you mention, but also what school is and being oriented to perspectives on learning.
You are right on that. I too believe that most habits are developed early on.
I guess I have a very different approach to how kids learn during elementary school.
I don't really think most of my education in elementary school came from the school itself. It was mostly from my parents (outside school activities like YMCA Summer camp, lots of arithmetic, etc.).
I think there's really no replacement to the local library at this age, and I don't think attending either public or private is of much aid on helping kids be encouraged to utilize the local library; I feel this role lies on the parents at this time period.

I would rather as an elementary kid go to a decent public school + have parents guiding me vs. going to a top private school with not as much aid from parents.
In my perspective, early on, parents have more impact on the kid's future than the school.
My parent always called me 'Doctor <Name>' (PhD type of doctor) early on to help build me confidence with my academics. And I recall early on in elementary school, my mother would turn on classical music in the house when eating breakfast while emphasizing the composers of those pieces (e.g.: Antonin Dvorák, Mozart, Haydn, Vivaldi, etc.) [in those days, there was a huge collection of classical music CDs just for this (+ Opera/Disney music CDs)] . My mother would talk about musicals (with sound on) like Les Miserables and so on.
My house had a kid's picture book version of famous people and stories (picture book [some abbreviated] versions of classics like Strange Case of Dr. Jekyll and Mr. Hyde, Aesop's Fables, The Little Prince, Isaac Newton, etc.).
And some educational typing games that helps kids get better with typing + learn new words.
Some comic books (manga format) about famous events/people/stories including the Three Kingdoms (China).
Having audiobooks on at night (I recall the Harry Potter audiobooks helped me sleep).
A lot of post tags with names of the furniture at home (e.g.: 'Refrigerator', 'Freezer' Post-it Notes on those items).
My parents would regularly converse with more advanced vocabulary at home (e.g.: regularly using words like daunting, immaculate, candid, etc.).
My mother would always ask what meal I want and give choices: 'Do you want Italian, Korean, Japanese? Okay, so what type of Italian food? Why?'
My parents encouraging to ask dumb questions like 'Why does X happen'.
My mother constantly pointing out flower names while on road trips.
My parents willing to adopt new ideas (while my parents grew up in Korea in which people around them always claimed 'you have to do X', my parents learnt a lot from traveling abroad that there's no 'fixed answer' to much of life).
My 'lullaby' (without me knowing) being the times table; '2 1 is 2, 2 2 is 4, 2 3 is 6, etc.' (it sounds really nice in Korean).
And of course, lots of vacation trips to different places each summer.
I also think I did a LOT of arithmetic. I think in some days, I did 3 books worth of problems in a given day when I was bored (each being almost 150 pages). Being good at basic arithmetic early on can really help. (all the parent has to do is 'assign the work' to be done for the day and mark the answers afterwards)
Never screaming/fighting/etc. at home. Parents always trying to reason out through talking and all.
Exposing me to all kinds of cuisines out there.
Studying with 3 other students of my year through having a small 'reading group' together with some exercises every week.
Having math puzzles (+ regular puzzles, fossil dig kits, etc.) at home to do lying on the side (sudokus are nice).
Collecting magnets of the different places my family vacationed and having all those magnets in a very visible collection (and parents constantly mentioning the good memories of those trips while pointing at the magnets during dinner time [almost always ate together]).
My mother having collections of Llado figurines, fancy teacups, etc. and pointing them out to me to help me appreciate the humanities (exposure helps).

Things I notice could be improved upon:
Maybe exposing kids to the names of different colors outside the basic 'red, yellow, orange, etc.'. Color names like magenta, cyan, etc.

I feel that actively participating parents (who have the money and time) are much better off investing into outside school activities than to spend a significant portion of that money on a private school instead (and be tight on budget for the outside school endeavors).
A lot of my 'passion' for math/computer science was built attending lego robotics classes and having done lots of math exercises early on (strong foundation --> good at school --> lots of praises --> enjoy the subject).
What I would like to know and was not addressed in your post is the network effect of private schools. Do you think network of a selective private school is more beneficial in later life compared to a good public school? How big of a an affect does a network have on future success and opportunities.
In terms of my own career, there was no 'network effect' with attending a private high school. Absolutely none (cause all my peers aren't even working in software engineering). It didn't matter that one of my peers came from one of the two wealthiest families in Korea. Nope. Not helpful at all.
In terms of friendships,... well, friends are friends. But there's no 'network' effect from my experience.
For college, I beg to differ. But that's outside the scope of this talk.

At end of day, each kid is different so there's no 'right answer'.
It's just my opinion that the money is better off being invested in outside school activities in the early years (with very active parents) as I feel early years of school is more to learn to socialize, etc. than to really 'study' academics.

Oh, and kids learn at different paces. Don't ever call them 'stupid', 'moron', etc.. Always make them feel like they can do anything.
I couldn't even iterate through all the months in the Calendar until 11th grade.
And videos like https://www.youtube.com/watch?v=0snNB1yS3IE are pretty entertaining. Spoken word poetry is nice to listen to from time to time. Maybe the child might be more interested in the humanities after listening to stuffs like this.
Also, try taking advantage of Khan Academy (maybe even PurpleMath). Kids can 'grind' points and finish a lot of contents fairly quick during weekends together with the parents.

[I'm sure my parents did so much more than just this. But I hope the list I made can be of some help for other parents here in Bogleheads. Once I have a family, I plan to do the same as my parents have done + more (if you haven't noticed, a lot of the activities can be 'expensive' quite quickly. If money is not a problem, then I guess you can do all this + private school but in general, with limited budget, I think active parent participation trumps an expensive private school in the early years)]
Also, I was kind of stuck in my mathematical progress post 8th grade cause Calculus I/II was the 'end' (at school, I would be taking AP Calc BC by 12th grade regardless). It would have been nice to have access to contents post Calculus I/II during school years. I learnt in college that many students attended their local colleges to take Differential Equations, Modern Algebra, etc. in high school. My parents didn't know such choices existed during high school.

Oh, and I did attend cram schools for some of these classes (math). Looking back, outside two teachers, the rest were waste of time/money. I wasted too much time here. Don't be peer pressured by others into sending your kid to 'X'. I ended up wasting my time attending SAT cram school from 8th to 11th and having my SAT practice score hover around (1940~2080)/2400 every summer [Math perfect score so you can assume the other 2 subject test scores]. Thankfully, my parents finally listened to me during end of junior year despite all the Korean brainwashing and let me self study (I moved back to Korea in my high school and my parents did get influenced a lot by the Asian peer pressure). Ended up raising the practice test scores to basically perfect score in 2 weeks of self study. I wish I could have all those summers back. Apparently, throwing money at a problem isn't always the solution.
I am fortunate that for majority of my school years, my parents let me be the ultimate decider of my learning. They were just there to guide. Sure they were highly pressured during high school due to the Korean peer pressure effect but ultimately, they decided to trust on my judgements more (and thank goodness). Each child is different and there will be kids that don't do well in private school learning. Please take note of this (paying more != better).

Also:
I noticed good privates tend to create two extremes. Either the student does really well or the student does really poorly. The students that didn't do as well lost all confidence and I have noted one of my peers literally have a seizure during class from the pressure. And a few students who left the private school ended up doing much better (two of them attended Harvard).
If I can redo elementary/middle/high school, I would choose a decent public school any day over a 'good' private. It wasn't for me too and the effects really showed in high school when I moved from a public middle school. Suddenly, my parents got peer pressured to all this fancy 'tutoring' nonsense and I stopped having as much good memories. I hated everything about the school starting from the strict uniforms (I couldn't take it).
In my personal experience, public schools don't create these kinds of extremes. So do take that into note.
Majority of my friends from my private high school agree if they can redo high school, they would always choose a decent public instead (something they would never admit to strangers). The stress/pressure/costs were not worth the experience. Fortunately, many of my friends are doing well today with one of them currently doing a Phd at Stanford. And almost all of them admitted that at end of day, self studying + encouraging home environment trumps throwing endless money to private tutoring.
halfnine
Posts: 2421
Joined: Tue Dec 21, 2010 12:48 pm

Re: Private School - Elementary vs Middle/High (bang for the buck)

Post by halfnine »

leeks wrote: Wed Jun 16, 2021 8:19 pm
halfnine wrote: Wed Jun 16, 2021 3:16 pm
leeks wrote: Tue Jun 15, 2021 5:57 pm
...An anecdote only - we are trying a private Montessori elementary next year for our children entering K and 2nd grades...
In general we wouldn't put our children in a Montessori school that typically allowed for non-Montessori transfer students especially beyond Kindergarten. Not if the reason we are paying is specifically for the Montessori environment. The schools that tend to allow transfers are often more "Montessori" in name than in function. That said, of course, the school could be perfectly fine, a great fit and a great education. I just wouldn't count on it being a true Montessori school.
I hear your concern. They are an accredited, well-established Montessori with properly trained teachers and such (a nonprofit overseen by a Board of Trustees). It is a relatively large school (up to 8th grade) and they do have more concern for achieving "challenging" academics than some Montessori programs (part of the appeal for us). They have three lower elementary (mixed grade 1-3) classrooms so there is always a bit of turnover (people move, can't afford it past a certain grade, etc) and they do fill empty spaces to keep roughly the same number of kids per grade.

There are fewer upper elementary and middle school classes though. I think a lot of parents find the most value in the early years but are willing to save money and switch to public as kids get older and the public offerings are more differentiated (there are upper elementary public gifted and specialty middle schools in the region). Although the pandemic changed that for some parents and there was talk of adding a third upper elementary class as more families wanted to stay next year.

But yeah, we had to prove our bona fides. They interview children and parents to determine academic levels and fit with Montessori style (they consistently have a waitlist so they get their pick of new students).

My mother was a Montessori teacher (I attended for preschool), I have been mostly a stay at home parent and our home life is reasonably consistent with Montessori values. My kids are self-driven, curious and (fiercely) independent. They can focus on a single task for a long time and they enjoy completing challenging things. We didn't have access to a Montessori preschool (they did half-day Reggio Emilia) but I feel they are well-suited to Montessori methods (and I realize that there will still be an adjustment phase).

What I can't know in advance is whether the school will live up to the reputation/promises and that the kids will be compatible with their specific teachers. That is a gamble as it would be with any school. We have our fingers crossed but I am willing to pull them out to temporarily homeschool again if it isn't as good a fit as we hope it will be.
halfnine wrote: Wed Jun 16, 2021 3:16 pm Disclosure: Our kids were in a Montessori school from ages 3 to 8.
Did your kids attend public schools after age 8?
Yes, they attend public schools. But it isn't because of some public/private debate it is simply because we live in an area where the public school is good enough fit for our children's needs. At least for now.

Ultimately at some point all Montessori kids have to transfer out of the Montessori environment. We felt the completion of Lower Elementary was a reasonable place for our circumstances. By then our kids had 6 years of Montessori and the Montessori ethos was well instilled within them. From a more academic standpoint they probably could have left at 7, however, we felt the leadership role typically afforded to the children during their last year in the mixed age group had enough value to keep them around for another year.
reader79
Posts: 98
Joined: Fri Jan 15, 2021 11:42 pm

Re: Private School - Elementary vs Middle/High (bang for the buck)

Post by reader79 »

I'm a semi-retired professor who attended both public and private schools. We have two small children, are highly educated, and have a high family household income. We wanted to rely on the data rather than anecdote when making this sort of decision, so we looked for large, longitudinal studies that included diverse students. The result: our kids attend public school because the evidence suggests that they will receive no additional benefit from a private education.

Here is an illustrative study on this question: https://www.publicschoolreview.com/blog ... ic-schools

The summary, which mirrors other studies, stands out:

"while private school students may be outperforming public school students, the difference is eliminated completely when you control for family income and parents’ level of educational achievement. Children birth through age 5 from high-income homes have educational resources that other children don’t get – conditions that are presumed to carry on through the child’s school years."

Of course, this is a personal decision that has no right or wrong answer. This is simply what we chose to do and why.
VTI: 50%, QQQM: 30%, VO: 10%, VB: 10%
oldfort
Posts: 2781
Joined: Mon Mar 02, 2020 7:45 pm

Re: Private School - Elementary vs Middle/High (bang for the buck)

Post by oldfort »

MMiroir wrote: Thu Jun 17, 2021 9:49 am
oldfort wrote: Wed Jun 16, 2021 10:04 pm
sdsu04 wrote: Tue Jun 15, 2021 5:20 pm 4. Do private schools prepare the child and provide a better opportunity to a higher ranked college (or a college of their choice)? Understand this is not a yes/no answer but your experience would be helpful to learn about and help us better understand their value.
There's an admissions advantage to attending a feeder school for admissions to the Ivies. 5% of Harvard freshman came from one of 7 schools. Feeder schools are usually private, but some, such as Stuyvesant, are public magnets.
https://www.thecrimson.com/article/2013 ... r-schools/
Many of those feeder schools remain feeders because alumni send their kids there. My daughter is at an Ivy, and knows six girls who were accepted from the same small all-girls schools in Connecticut. All were legacies. So the school is a great feeder if you are a legacy, not so much if you are not.
If you don't have some hook: athlete, legacy, race or ethnicity, dean's interest list, or child of faculty or staff, the most selective Ivies are a long shot anyway.
oldfort
Posts: 2781
Joined: Mon Mar 02, 2020 7:45 pm

Re: Private School - Elementary vs Middle/High (bang for the buck)

Post by oldfort »

reader79 wrote: Thu Jun 17, 2021 3:45 pm I'm a semi-retired professor who attended both public and private schools. We have two small children, are highly educated, and have a high family household income. We wanted to rely on the data rather than anecdote when making this sort of decision, so we looked for large, longitudinal studies that included diverse students. The result: our kids attend public school because the evidence suggests that they will receive no additional benefit from a private education.

Here is an illustrative study on this question: https://www.publicschoolreview.com/blog ... ic-schools

The summary, which mirrors other studies, stands out:

"while private school students may be outperforming public school students, the difference is eliminated completely when you control for family income and parents’ level of educational achievement. Children birth through age 5 from high-income homes have educational resources that other children don’t get – conditions that are presumed to carry on through the child’s school years."

Of course, this is a personal decision that has no right or wrong answer. This is simply what we chose to do and why.
The family characteristics used as control variables in their model were:
Family characteristics. Our first block of covariates taps into families’ capacity and resources, namely: mothers’ age and years of education at birth of child, mothers’ psychological adjustment at 6 months of age (measured with the NEO Personality Inventory; Costa & McCrae, 1985), mothers’ vocabulary skills at 36 months of age (measured with the Peabody Picture Vocabulary Test–Revised; Dunn & Dunn, 1981), parenting quality (measured with the Home Observation for Measurement of the Environment Scale, Caldwell & Bradley, 1984; and videotaped interactions that captured maternal sensitivity), household income-to-needs ratio, maternal employment, maternal depressive symptoms (measured with the Center for Epidemiological Studies Depression Scale; Radloff, 1977), and an indicator of whether children lived in a two-parent household.
As with all studies, what the authors choose to include or exclude as control variables, tells you a lot about their view of the world.
Post Reply